Given that a rectangle has a length of 5/2x + 10 with a width of 5/2x + 5, formulate an expression to represents the area of the rectangle.

Answers

Answer 1

Answer:

              A =  ²⁵/₄x² + ⁷⁵/₂x + 50

Step-by-step explanation:

L = ⁵/₂x + 10

W = ⁵/₂x + 5

A = L•W

A = (⁵/₂x + 10)(⁵/₂x + 5)

A = ⁵/₂x•⁵/₂x + ⁵/₂x•5 + 10•⁵/₂x + 10•5

A = ²⁵/₄x² + ²⁵/₂x + ⁵⁰/₂x + 50

A =  ²⁵/₄x² + ⁷⁵/₂x + 50

Or if yoy mean:

L = 5/(2x) + 10

W = 5/(2x) + 5

A = [5/(2x) + 10][5/(2x) + 5] = 25/(4x²) + 75/(2x) + 50


Related Questions

What is another name for line P?

Answers

Answer:

Line CD

Step-by-step explanation:

The another name for line P is line [tex]\over (CD)[/tex].

It is required to find the  another name for line P.

What is line?

A line is a straight one-dimensional figure having no thickness and extending infinitely in both directions. A line is made of a set of points which is extended in opposite directions infinitely. It is determined by two points in a two-dimensional plane. The two points which lie on the same line are said to be collinear points.

Given:

To answer this type of problem we have to know the axiom.

At least two points define a line.

Then I can call line p as (see below attached picture)

The line p cuts the 2 different points , point C and point D, so we can.t indicate a line in a single letter of the Western Alphabet.

Therefore, the another name for line P is line [tex]\over (CD)[/tex].

Learn more about line here:

brainly.com/question/17310681

#SPJ2

Solve 3[-x + (2 x + 1)] = x - 1.
x=

Answers

Answer:

x= -2

Step-by-step explanation:

PLS HELLP!!!!!!!!!!!!!!!!!!!!! Deandra is doing her math homework. Out of 24 problems total, she completed the first 12 problems in 1 hour and the last 12 problems in 3 hours. What is the unit rate for all 24 problems?

Answers

Answer:

24

Step-by-step explanation:

PLEASE HELP ME ASP!!!!!!

Answers

Answer:

B

Step-by-step explanation:

First, note that the figure is a parallelogram.

The area of a parallelogram is given by:

[tex]A=bh[/tex]

Where b is the base length and h is the vertical height.

The base length is 5+2 or 7, and the vertical height is 2.

Thus:

[tex]A=7(2)\\A=14\text{ in}^2[/tex]

The answer is B.

Which pair of numbers has 7 as its least common multiple? Group of answer choices 1, 7 3, 4 7, 21 14, 28

Answers

Answer:

7, 14, 14,21, and 28

Step-by-step explanation:

Only 1, 7 is pair of numbers that has 7 as its least common multiple.

Given that,
Pairs ( 1, 7 ), ( 3, 4 ), ( 7, 21), ( 14, 28).
A pair or pairs to be determined that have 7 as its least common multiple.

What is arithmetic?

In mathematics, it deals with numbers of operations according to the statements.

What is Lowest Common Multiple?

In arithmetic, LCM is defined as the lowest common multiple of two integers X and Y, usually represented as by LCM X, Y, and is the smallest favorable integer that is dividable by both X and Y.

Since the Least common multiple of pair of numbers has 7, this is only possible for 1 * 7, because the LCM of 1 and7 is 7. So from given option 1, 7 is the required pair.

Thus, only 1, 7 is pair of numbers that has 7 as its least common multiple.

Learn more about arithmetic here:

brainly.com/question/14753192

#SPJ2


Tavon has a gift card for $165 that loses $4 for each 30-day period it is not used. He has another gift card for $145 that loses $3.50 for each 30-day period it is not used. Write an equation for the number of 30-day periods until the value of the gift cards will be equal. Let x represent the number of 30-day periods.

Answers

Answer:

x=20

Step-by-step explanation:

Let  = a 30 day period

The first equation is 50 - 2x (because you start with $50 and subtract 2 for each 30 day period)

The second equation is 40-1.5x

Set them equal to each other:  50 - 2x = 40 - 1.5x

Solve for x:

50 - 2x = 40 - 1.5 x

    +2x         +2x

-----------------------

50 = 40 + 0.5 x

-40   -40

-------------------

10 = 0.5 x

20 = x

So there will be 20 30-day periods until the cards will be equal in value.

Check:  After 20 periods, 50 - 2(20) = $10 left

40 - 1.5(20) = also $10 left

Imagine that you earned $8,425 in one year. If the government enforces a 15% income tax, how much money would you owe in taxes at the end of the year?

Answers

Answer: You would owe $1,263 and 75c! Have a fantastic day!

express 0.000834 in standard form​

Answers

Answer:8.34 x 10-4

                                      mark me brainliest

Answer:8.34 x 10 -4 is the answer

These box plots show daily low temperatures for a sample of days in two
different towns.
Town A
10 15 20
30
55
Town B
5
20
30
40
55
0
5
10
15
20
45
50
55
60
25 30 35 40
Degrees (F)
Compare the shapes of the box plots.
O A. The distribution for town A is negatively skewed, but the
distribution for town B is symmetric.
B. The distributions for both towns are positively skewed.
C. The distributions for both towns are symmetric.
O D. The distribution for town A is positively skewed, but the
distribution for town B is symmetric.

Answers

D. The distribution for town A is positively skewed, but the

distribution for town B is symmetric.

The distribution for town A is positively skewed, but the distribution for town B is symmetric. The correct option is D.

What is box plots?

Box plots, also known as box-and-whisker plots, are a way to visualize the distribution of a set of numerical data.

To compare the shapes of the box plots, we need to look at the position of the median, the length of the box, and the length of the whiskers.

In the box plot for town A, the median is closer to the bottom of the box, indicating that the lower half of the data is more spread out than the upper half.

The whisker on the left side of the box plot is longer than the whisker on the right side, which indicates that there are some extreme values on the lower end of the data. This is consistent with a positively skewed distribution.

In contrast, the box plot for town B has a symmetrical box, with the median located near the center.

The whiskers on both sides of the box plot are roughly the same length, indicating that there are no extreme values on either end of the data. This is consistent with a symmetric distribution.

Thus, the correct option is D.

For more details regarding box plots, visit:

https://brainly.com/question/1523909

#SPJ5

Evaluate a) 3⁴ b) -6² c) (-4)⁴

Answers

Answer:

A) 81

B)-36

C)256

Step-by-step explanation:

A)

We have:

[tex]3^4[/tex]

This is the same as:

[tex]=3\times3\times3\times3[/tex]

This equals:

[tex]=81[/tex]

B)

We have:

[tex]-6^2[/tex]

This one is tricky. Note that the negative is not tied to 6. There's no parentheses. This is the same as saying:

[tex]=(-1)(6)^2[/tex]

Thus, evaluate the square:

[tex]=-1(36)[/tex]

Multiply:

[tex]=-36[/tex]

C)

We have:

[tex](-4)^4[/tex]

This is the same as:

[tex]=-4\times-4\times-4\times-4[/tex]

Multiply:

[tex]=256[/tex]

Answer:

A. (3x4=12)

B. (-12)

C. ( -16

Step-by-step explanation:

Hope it helps out.

3x4=12    -6x2=-12     -4x4=-16

What is the perimeter of a rectangle with length of √18 and width of √8 in simplest form?

Answers

Answer:

P=10√2

Step-by-step explanation:

Perimeter of rectangle= 2 length + 2 width

P=2l+2w

P=2(√18)+2(√8)

P=2(√9*2)+2(√4*2)

P=2*3*√2 +2*2√2

P=6√2+4√2

P=10√2

Solve for x. 2x + 2 = -8 X=???? Please help!?

Answers

Answer:

x = - 5

Step-by-step explanation:

2x + 2 = - 8

2x = - 8 - 2

2x = - 10

x = - 10 : 2

x = - 5

Answer:

x=-5

Step-by-step explanation:

1.) 2x+2=-8   start to isolate the variable by subtracting 2 to -8

2.) 2x=-10 divide both sides by 2

3.) x=-5

Hope this helped

Complete the sequence,
13, 14, 27, 41,

Answers

Answer:

68

Step-by-step explanation:

We can see that the nth term would be the sum of the previous two terms. Continuing this would result in 68.

13. 4 times the difference of a and
b

Answers

Answer: 4(a - b)

Step-by-step explanation:

If 4 is multiply by the difference of  a and b  then we will have the expression,

4(a-b)

How to Multiply six and the reciprocal of the quotient of a number and seven?

Answers

Answer:

Let N be the unknown number

The we have 6 x reciprocal of N/7  which is 6 x 7/N which is 42/N

HOPE THIS HELPS YOU

Multiply six and the reciprocal of the quotient of a number and seven can be written as 42/M.

Where M is the number.

Given,

Multiply six and the reciprocal of the quotient of a number and seven.

We need to find this expression.

What does quotient mean?

It means to divide.

The quotient of a and b:

=  a ÷ b.

= a / b

Find the expression.

Let the number be M.

Now,

The quotient of a number and 7 can be written as:

= M ÷ 7

= M / 7

Reciprocal of the quotient of a number and seven.

= 1 / (M/7)

= 7 / M

Now,

Multiply six and the reciprocal of the quotient of a number and seven.

It can be written as:

= 6 x 7 / M

= 42 / M

Thus multiply six and the reciprocal of the quotient of a number and seven can be written as 42/M.

Learn more about the quotient of a number and its reciprocal here:

https://brainly.com/question/673545

#SPJ2

On January 1st 2020 the populations of Japan and the Philippines were 126.5million and 109.6 respectively. Japan's population is currently decreasing by 0.2% per year while the population of the Philippines is increasing by 1.6% per year. If these rates of change remain constant, during which year will the two population be the same​

Answers

Well that will happen I’m not sure I think 2040

In 422.5 years, the two populations will be the same

Let the year when the two populations will be the same be represented by x.

The equation to solve the question will be:

126.5 - 0.2x = 109.6 - 0.16x

Collect like terms

0.2x - 0.16x = 126.5 - 109.6

0.04x = 16.9

x = 16.9/0.04

x = 422.5

In conclusion, in 422.5 years, the two populations will be the same.

Read related link on:

https://brainly.com/question/17723234

Find the cosine ratio of angle 0

Answers

Answer: cos(0) = 1

hope this helped

The regular price of a child's entry ticket to a water park is $8 less than that for an adult's. The park offers half off all entry tickets during the off-peak season. The Sandlers paid a total of $194 for 1 adult ticket and 4 child's tickets to the water park during the off-peak season. The following equation represents this situation, where x represents the regular price of an adult ticket:

194 = one-half x + 2(x − 8)

What is the regular price of a child's ticket? (5 points)

a
$68

b
$76

c
$84

d
$89

Answers

Answer:

76

Step-by-step explanation:

so first the equation for the situation has been given which is

194=1/2x + 2x - 16

now you can just solve since it's a linear equation

194=2.5x -16

take 16 to the other side to get

210=2.5x

solve to get x (adult price)

which is 84

remember the. child's price is 8 less

84-8=76

∠A and \angle B∠B are complementary angles. If m\angle A=(2x+10)^{\circ}∠A=(2x+10) ∘ and m\angle B=(3x+15)^{\circ}∠B=(3x+15) ∘ , then find the measure of \angle B∠B.

Answers

Answer:

[tex]54^\circ[/tex]

Step-by-step explanation:

Given :

[tex]m\angle A=(2x+10)^{\circ}[/tex]

[tex]m\angle B=(3x+15)^{\circ}[/tex]

And angles [tex]\angle A[/tex] and [tex]\angle B[/tex] are complementary angles.

To find:

The measure of [tex]\angle B[/tex].

Solution:

First of all, let us learn about complementary angles.

Complementary angles are the angles whose sum is equal to [tex]90^\circ[/tex].

And we are given that angles [tex]\angle A[/tex] and [tex]\angle B[/tex] are complementary angles.

Therefore [tex]\angle A[/tex] + [tex]\angle B[/tex] = [tex]90^\circ[/tex]

[tex]\Rightarrow 2x+10+3x+15=90\\\Rightarrow 5x+25=90\\\Rightarrow 5x=65\\\Rightarrow x =13[/tex]

Putting the value in [tex]\angle B[/tex].

[tex]m\angle B=(3\times 13+15)^{\circ} = 54^\circ[/tex]

Therefore, the answer is [tex]\bold{\angle B = 54^\circ}[/tex]

please help i need this answered as soon as possible!!! -Use ΔDEF, shown below, to answer the question that follows:

Answers

Answer:

X= 139

Step-by-step explanation:

the square on the E means its a right side so that would be 90 degrees.  All triangles add up to 180.  F would be 41 degrees.  that mean X is  139 i think.

Answer:

36.083

Step-by-step explanation:

SOHCAHTOA

Cosine is adjacent over hypotenuse, so the

cos(49) = the adjacent side (x) || over the hypotenuse (55)

cos(49) = x/55

55cos(49) = x

x = 36.083

From the set {1, 2, 3, 4, 5}, find the lowest value of x that makes each inequality true. Arrange the inequalities in increasing order of the lowest value of x from the set that makes them true. 5x − 3x > 9 2x − 1 > 0 x + (x − 1) > 5

Answers

2x - 1 > 0 ( x = 1)

x + (x − 1) > 5 ( x = 3)

5x − 3x > 9 ( x = 5)

Answer:

2x - 1 > 0 ( x = 1)

x + (x − 1) > 5 ( x = 3)

5x − 3x > 9 ( x = 5)Step-by-step explanation:

Four more than twice a number is –10.

Answers

Four more than twice a number is –10 is the same as 2n + 4 = 10

Steps to solve:

2n + 4 = 10

~Subtract 4 to both sides

2n = 6

~Divide 2 to both sides

n = 3

Best of Luck!

Aieeeee I was looking for an answer to this

Consider the line 9x - 5y = 2
What is the slope of a line parallel to this line?
What is the slope of a line perpendicular to this line?

Answers

Answer:

5y=9x-2

y=mx-b

m=slope

y=9/5x-2/5

m=9/5=1.8 slope of pArallel

slope of pARALLEL=-1/M SLOPE OF A LINE PERPENDICULAR

m=-5/9 slope of a line perpendiculr


Angle X and Y are supplementary. The measure of the angle X is 19° less than 2 times the measure of angle Y. What is the
measure of angle x?

Answers

Answer:

Supplementary angles add up to 180 degrees. The key phrase "less than" signifies you are going to subtract angle X from angle Y. Angle Y is 2 times the measure of angle X so you are going to multiply Y by 2. So right now you should have the equation 2y - 19 = 180. Then, plug it in. Add 19 to 180 to move it to the other side and then divide by 2.

Which function has a domain of x 25 and a range of y s3?

Answers

Answer:

Step-by-step explanation:

The function  has a domain x ≥ 5.

This is because the function remains real for (x - 5) ≥ 0 as negative within the square root is imaginary.

Hence, (x - 5) ≥ 0

⇒ x ≥ 5

Now, for all x values that are greater than equal to 5 the value of  will be negative.

So,  

⇒  

⇒ y ≤ 3

Therefore, the range of the function is y ≤ 3. (Answer)

Note: Thanks rani

what is 38.61 + 36.481

Answers

The answer to that question is 75.091

Answer:

75.091

Step-by-step explanation:

What two rational expressions sum to 2x+3/x^2-5x+4

Enter your answer by filling in the boxes. Enter your answer so that each rational expression is in simplified form.

Answers

Answer:

[tex] \frac{2x + 3}{(x- 1)(x - 4)} = \frac{-5}{3(x- 1)} + \frac{11}{3(x - 4)} [/tex]

Step-by-step explanation:

Given the rational expression: [tex] \frac{2x + 3}{x^2 - 5x + 4} [/tex], to express this in simplified form, we would need to apply the concept of partial fraction.

Step 1: factorise the denominator

[tex] x^2 - 5x + 4 [/tex]

[tex] x^2 - 4x - x + 4 [/tex]

[tex] (x^2 - 4x) - (x + 4) [/tex]

[tex] x(x - 4) - 1(x - 4) [/tex]

[tex] (x- 1)(x - 4) [/tex]

Thus, we now have: [tex] \frac{2x + 3}{(x- 1)(x - 4)} [/tex]

Step 2: Apply the concept of Partial Fraction

Let,

[tex] \frac{2x + 3}{(x- 1)(x - 4)} [/tex] = [tex] \frac{A}{x- 1} + \frac{B}{x - 4} [/tex]

Multiply both sides by (x - 1)(x - 4)

[tex] \frac{2x + 3}{(x- 1)(x - 4)} * (x - 1)(x - 4) [/tex] = [tex] (\frac{A}{x- 1} + \frac{B}{x - 4}) * (x - 1)(x - 4) [/tex]

[tex] 2x + 3 = A(x - 4) + B(x - 1) [/tex]

Step 3:

Substituting x = 4 in [tex] 2x + 3 = A(x - 4) + B(x - 1) [/tex]

[tex] 2(4) + 3 = A(4 - 4) + B(4 - 1) [/tex]

[tex] 8 + 3 = A(0) + B(3) [/tex]

[tex] 11 = 3B [/tex]

[tex] \frac{11}{3} = B [/tex]

[tex] B = \frac{11}{3} [/tex]

Substituting x = 1 in [tex] 2x + 3 = A(x - 4) + B(x - 1) [/tex]

[tex] 2(1) + 3 = A(1 - 4) + B(1 - 1) [/tex]

[tex] 2 + 3 = A(-3) + B(0) [/tex]

[tex] 5 = -3A [/tex]

[tex] \frac{5}{-3} = \frac{-3A}{-3} [/tex]

[tex] A = -\frac{5}{3} [/tex]

Step 4: Plug in the values of A and B into the original equation in step 2

[tex] \frac{2x + 3}{(x- 1)(x - 4)} = \frac{A}{x- 1} + \frac{B}{x - 4} [/tex]

[tex] \frac{2x + 3}{(x- 1)(x - 4)} = \frac{-5}{3(x- 1)} + \frac{11}{3(x - 4)} [/tex]

Answer:

Step-by-step explanation:

Given the rational expression: , to express this in simplified form, we would need to apply the concept of partial fraction.

Step 1: factorise the denominator

Thus, we now have:

Step 2: Apply the concept of Partial Fraction

Let,

=

Multiply both sides by (x - 1)(x - 4)

=

Step 3:

Substituting x = 4 in

Substituting x = 1 in

Step 4: Plug in the values of A and B into the original equation in step 2

Step-by-step explanation:

The volume, in cubic feet, of a right cylindrical silo of height and radius is [tex]V = \pi r^2h[/tex] The height of the silo is h(r) = 3.5r. Which statements are true regarding the functions described?


A. To get a volume of 100 cubic feet, the radius must be 2 feet.


B. The domain of [tex]V(h(r))[/tex] is restricted to values of r greater than 0.


C. The output of V is the input of h


D. [tex]V(h(r)) = 3.5\pi r^3[/tex]


E. The volume depends on the radius of the cylinder

Answers

1. The domain of [tex]V (h(r))[/tex] is restricted to values of r greater than 0.

2. [tex]V(h(r)) = 3.5\pi r^3[/tex]

3. The volume depends on the radius of the cylinder

How to round 1724.89 to the nearest tenths place ?

Answers

Answer:

1724.9

Step-by-step explanation:

1724.89 ≈ 1724.9

Answer:

1724.9

Step-by-step explanation:

If you want to round to tenths place, look at hundredths place.

If the digit in  hundredths place is  5 or more than 5  (5,6,7,8,9), then add '1' to the tenth place.

Here, 9 is in hundredths place, so add 1 to 8

1724.9

Given: 2x + 3(x - 2) = 24
Prove: x = 6
Statements and Reasons?

Answers

Answer:

x=6

Step-by-step explanation:

first we'll use distributive property: we will distribute and multiply 3 by x and - 2

2x + 3x -6 = 24

then we'll add like terms which are 2x and 3x , 24 and 6 the sign of 6 will change from negative to positive:

2x + 3x =24 + 6

5x = 30

after that we will divide both sides by 5 in order to get x

5x/5 = 30/ 5

x = 6

Other Questions
A CAR IS TRAVELING ON A FREEWAY AT 50 MPH WITH THE CRUISE CONTROL SET AT 50 MPH. ANOTHER CAR IS TRAVELING AT 90 MPH WITH THE CRUISE CONTROL SET AT 90 MPH. WHICH CAR HAS A HIGHER ACCELERATION? In which order are the following measuring instruments listed from lowest precision to highest precision:Select one:a. Outside Calipers. Ruler. Micrometer. Digital Calipers.b. Outside Calipers.Digital Calipers. Ruler. Micrometer.c. Outside Calipers.Digital Calipers. Micrometer. Ruler. who ruled in gorkha Can someone please help me with 4+5 and 6 pleaseeeee Qu opinas t sobre el tema de la discriminacin racial? porfis nesesito ayuda con esa tarea List the new coordinate of point J' when the figure is reflected over the y-axis. Hi can someone please,please,please help me convert 56 inches to centimeters pleaseee show steps according to the proclamation of 1763 at the end of the french and indian wat Simplify 12 to the 16th power over 12 to the 4th power. Samanthas savings account has a balance of $4643. After 25 years, what will the amount of interest be at 6% compounded annually? Legendary stories express the underlying beliefs and values of those who told them and shape our vision of the past True or false ? whats the tone of this sentence the day was dark and cloudy. i walked to school in the rain. my homework got all wet. What happened????????? While John is traveling along a straight inter-state highway, he notices that the mile markerreads 241 km. John travels until he reachesthe 145 km marker and then retraces his pathto the 174 km marker.What is John's resultant displacement fromthe 241 km marker?Answer in units of km. The graph shows Ms. Collin's driving speed during 10 minutes of her commute. Driving Speed 70 60+ 50 40 Speed (miles per hour) 30 20 10 0 1 2 9 10 3 4 5 6 7 8 Time (minutes) Between which times did Ms. Collin drive at a constant speed? A. O and 2 minutes B. 2 and 4 minutes O c. 4 and 6 minutes D. 6 and 8 minutes Yosemite Falls, in California, has a total height of 73,000cm. What is the height in meters? g The false positive rate, P(+|N), for a test is given as 0.04. What is the specificity for this test? If there are 17.5 grams of sugar in 225 grams of an aqueous sugar solution, the percent-mass sugar is ______ and the percent-mass water is ______ . Express both answers to the correct number of significant figures. PLEASE HELP! 20 question